Five Powers of Fives Produce Unique Pandigital Number…Solve for X..Tell me YCrack the Code #1Longest Calculator Word?A rather curious division machineVegas Street Magician Math TrickPassword CrackingFind a Strobogrammatic number, so if we square it, the result is a pandigit numberHoneydripping around the clockA mo-Roman samplerFind the equality with all digitsLong digital sequence. 16xxxxxxxxxxxxx61

Is "vegetable base" a common term in English?

Storing voxels for a voxel Engine in C++

The disk image is 497GB smaller than the target device

Unary Enumeration

One word for 'the thing that attracts me'?

Magento2: How can call observer function in controller file?

Why did it take so long for Germany to allow electric scooters / e-rollers on the roads?

Shell script fails to create a mysql backup

Status of proof by contradiction and excluded middle throughout the history of mathematics?

How to teach an undergraduate course without having taken that course formally before?

How to capitalise every letter in odd position as in memes?

Why does FOO=bar; export the variable into my environment

Why does the hash of infinity have the digits of π?

How to query/filter by the value of a lightswitch

How would a developer who mostly fixed bugs for years at a company call out their contributions in their CV?

Python program for fibonacci sequence using a recursive function

Are PMR446 walkie-talkies legal in Switzerland?

Is it normal to "extract a paper" from a master thesis?

Are cells guaranteed to get at least one mitochondrion when they divide?

What could be my risk mitigation strategies if my client wants to contract UAT?

Question about Shemot, locusts

3 prong range outlet

Why'd a rational buyer offer to buy with no conditions precedent?

Can a multiclassed Kensei monk/Swashbuckler rogue use an offhand finesse weapon to trigger Sneak Attack, without using a bonus action?



Five Powers of Fives Produce Unique Pandigital Number…Solve for X..Tell me Y


Crack the Code #1Longest Calculator Word?A rather curious division machineVegas Street Magician Math TrickPassword CrackingFind a Strobogrammatic number, so if we square it, the result is a pandigit numberHoneydripping around the clockA mo-Roman samplerFind the equality with all digitsLong digital sequence. 16xxxxxxxxxxxxx61













11












$begingroup$


Given: Y is a Pan-digital Number (no zero, 1 to 9 only) ending in 3.



Pan digital number consists of all 9 digits 1 to 9..each digit occurring only once as is the case here. Last digit is given as 3 and all other digits 1,2,4,5,6,7,8,9 can be anywhere in the number.



No googling, no computers, you don’t even need the calculator till the last step to calculate Y from X.



$Y = (X-1) ^ 5 + ( X + 7 ) ^ 5 + ( 2X + 6 ) ^ 5 + ( 4X + 3 ) ^ 5 + ( 5 X + 8) ^ 5$



The most concise and logical answer will be accepted.










share|improve this question











$endgroup$







  • 2




    $begingroup$
    In the future, for these kinds of alphametic/mathematical puzzles, I strongly encourage you to use MathJax to make the formatting look good, as it does now. Here's a meta post from Mathematics SE that gives a quick tutorial. I also recommend reading the Markdown help page for other formatting matters. Keep up the good work! :)
    $endgroup$
    – PiIsNot3
    May 16 at 3:41










  • $begingroup$
    You may add the definition of pan-digital number (or put a link will do), I just know that term today
    $endgroup$
    – athin
    May 16 at 3:42










  • $begingroup$
    Thx..will do in the future..didn’t have time to learn it fully
    $endgroup$
    – Uvc
    May 16 at 3:43










  • $begingroup$
    Has a correct answer been given? If so, please don't forget to $colorgreencheckmark smalltextAccept$ it :)
    $endgroup$
    – Rubio
    yesterday










  • $begingroup$
    Explanations and answers given are very elaborate. What is required is concise logical answer. Once given, it will be accepted. I think it can be done with less than 100 characters easily.
    $endgroup$
    – Uvc
    yesterday
















11












$begingroup$


Given: Y is a Pan-digital Number (no zero, 1 to 9 only) ending in 3.



Pan digital number consists of all 9 digits 1 to 9..each digit occurring only once as is the case here. Last digit is given as 3 and all other digits 1,2,4,5,6,7,8,9 can be anywhere in the number.



No googling, no computers, you don’t even need the calculator till the last step to calculate Y from X.



$Y = (X-1) ^ 5 + ( X + 7 ) ^ 5 + ( 2X + 6 ) ^ 5 + ( 4X + 3 ) ^ 5 + ( 5 X + 8) ^ 5$



The most concise and logical answer will be accepted.










share|improve this question











$endgroup$







  • 2




    $begingroup$
    In the future, for these kinds of alphametic/mathematical puzzles, I strongly encourage you to use MathJax to make the formatting look good, as it does now. Here's a meta post from Mathematics SE that gives a quick tutorial. I also recommend reading the Markdown help page for other formatting matters. Keep up the good work! :)
    $endgroup$
    – PiIsNot3
    May 16 at 3:41










  • $begingroup$
    You may add the definition of pan-digital number (or put a link will do), I just know that term today
    $endgroup$
    – athin
    May 16 at 3:42










  • $begingroup$
    Thx..will do in the future..didn’t have time to learn it fully
    $endgroup$
    – Uvc
    May 16 at 3:43










  • $begingroup$
    Has a correct answer been given? If so, please don't forget to $colorgreencheckmark smalltextAccept$ it :)
    $endgroup$
    – Rubio
    yesterday










  • $begingroup$
    Explanations and answers given are very elaborate. What is required is concise logical answer. Once given, it will be accepted. I think it can be done with less than 100 characters easily.
    $endgroup$
    – Uvc
    yesterday














11












11








11





$begingroup$


Given: Y is a Pan-digital Number (no zero, 1 to 9 only) ending in 3.



Pan digital number consists of all 9 digits 1 to 9..each digit occurring only once as is the case here. Last digit is given as 3 and all other digits 1,2,4,5,6,7,8,9 can be anywhere in the number.



No googling, no computers, you don’t even need the calculator till the last step to calculate Y from X.



$Y = (X-1) ^ 5 + ( X + 7 ) ^ 5 + ( 2X + 6 ) ^ 5 + ( 4X + 3 ) ^ 5 + ( 5 X + 8) ^ 5$



The most concise and logical answer will be accepted.










share|improve this question











$endgroup$




Given: Y is a Pan-digital Number (no zero, 1 to 9 only) ending in 3.



Pan digital number consists of all 9 digits 1 to 9..each digit occurring only once as is the case here. Last digit is given as 3 and all other digits 1,2,4,5,6,7,8,9 can be anywhere in the number.



No googling, no computers, you don’t even need the calculator till the last step to calculate Y from X.



$Y = (X-1) ^ 5 + ( X + 7 ) ^ 5 + ( 2X + 6 ) ^ 5 + ( 4X + 3 ) ^ 5 + ( 5 X + 8) ^ 5$



The most concise and logical answer will be accepted.







mathematics no-computers






share|improve this question















share|improve this question













share|improve this question




share|improve this question








edited May 16 at 8:03









Community

1




1










asked May 16 at 3:34









UvcUvc

83113




83113







  • 2




    $begingroup$
    In the future, for these kinds of alphametic/mathematical puzzles, I strongly encourage you to use MathJax to make the formatting look good, as it does now. Here's a meta post from Mathematics SE that gives a quick tutorial. I also recommend reading the Markdown help page for other formatting matters. Keep up the good work! :)
    $endgroup$
    – PiIsNot3
    May 16 at 3:41










  • $begingroup$
    You may add the definition of pan-digital number (or put a link will do), I just know that term today
    $endgroup$
    – athin
    May 16 at 3:42










  • $begingroup$
    Thx..will do in the future..didn’t have time to learn it fully
    $endgroup$
    – Uvc
    May 16 at 3:43










  • $begingroup$
    Has a correct answer been given? If so, please don't forget to $colorgreencheckmark smalltextAccept$ it :)
    $endgroup$
    – Rubio
    yesterday










  • $begingroup$
    Explanations and answers given are very elaborate. What is required is concise logical answer. Once given, it will be accepted. I think it can be done with less than 100 characters easily.
    $endgroup$
    – Uvc
    yesterday













  • 2




    $begingroup$
    In the future, for these kinds of alphametic/mathematical puzzles, I strongly encourage you to use MathJax to make the formatting look good, as it does now. Here's a meta post from Mathematics SE that gives a quick tutorial. I also recommend reading the Markdown help page for other formatting matters. Keep up the good work! :)
    $endgroup$
    – PiIsNot3
    May 16 at 3:41










  • $begingroup$
    You may add the definition of pan-digital number (or put a link will do), I just know that term today
    $endgroup$
    – athin
    May 16 at 3:42










  • $begingroup$
    Thx..will do in the future..didn’t have time to learn it fully
    $endgroup$
    – Uvc
    May 16 at 3:43










  • $begingroup$
    Has a correct answer been given? If so, please don't forget to $colorgreencheckmark smalltextAccept$ it :)
    $endgroup$
    – Rubio
    yesterday










  • $begingroup$
    Explanations and answers given are very elaborate. What is required is concise logical answer. Once given, it will be accepted. I think it can be done with less than 100 characters easily.
    $endgroup$
    – Uvc
    yesterday








2




2




$begingroup$
In the future, for these kinds of alphametic/mathematical puzzles, I strongly encourage you to use MathJax to make the formatting look good, as it does now. Here's a meta post from Mathematics SE that gives a quick tutorial. I also recommend reading the Markdown help page for other formatting matters. Keep up the good work! :)
$endgroup$
– PiIsNot3
May 16 at 3:41




$begingroup$
In the future, for these kinds of alphametic/mathematical puzzles, I strongly encourage you to use MathJax to make the formatting look good, as it does now. Here's a meta post from Mathematics SE that gives a quick tutorial. I also recommend reading the Markdown help page for other formatting matters. Keep up the good work! :)
$endgroup$
– PiIsNot3
May 16 at 3:41












$begingroup$
You may add the definition of pan-digital number (or put a link will do), I just know that term today
$endgroup$
– athin
May 16 at 3:42




$begingroup$
You may add the definition of pan-digital number (or put a link will do), I just know that term today
$endgroup$
– athin
May 16 at 3:42












$begingroup$
Thx..will do in the future..didn’t have time to learn it fully
$endgroup$
– Uvc
May 16 at 3:43




$begingroup$
Thx..will do in the future..didn’t have time to learn it fully
$endgroup$
– Uvc
May 16 at 3:43












$begingroup$
Has a correct answer been given? If so, please don't forget to $colorgreencheckmark smalltextAccept$ it :)
$endgroup$
– Rubio
yesterday




$begingroup$
Has a correct answer been given? If so, please don't forget to $colorgreencheckmark smalltextAccept$ it :)
$endgroup$
– Rubio
yesterday












$begingroup$
Explanations and answers given are very elaborate. What is required is concise logical answer. Once given, it will be accepted. I think it can be done with less than 100 characters easily.
$endgroup$
– Uvc
yesterday





$begingroup$
Explanations and answers given are very elaborate. What is required is concise logical answer. Once given, it will be accepted. I think it can be done with less than 100 characters easily.
$endgroup$
– Uvc
yesterday











3 Answers
3






active

oldest

votes


















7












$begingroup$

OK, now i realise its beauty...




for all $i$ from $0$ to $9$, $i^5$ mod $10$ = $i$




so to simply get the ending digit of $X$, the equation can be simplified:




$Y = (X-1) + ( X + 7 ) + ( 2X + 6 ) + ( 4X + 3 ) + ( 5 X + 8) $
$Y=13X +23$




More mod10-ing:




$Y=3X+3$




Sub $Y mod10 = 3$:




$3=3X+3$




Deducing $X mod10$:




$3X=0$
$X=0 (mod 10)$




Then, start from $X = $




$0$




Result (using a calculator in this very last step)




$57593$ Too small...




Next attempt: $X =$




$10$




Result (using a calculator in this very last step)




$Y=816725493$ Nice!




The result above is a pan-digital number as required by OP, so this is done!






share|improve this answer











$endgroup$












  • $begingroup$
    ...yes.........
    $endgroup$
    – Uvc
    May 16 at 4:08










  • $begingroup$
    beautiful question!!! @Uvc +1ed
    $endgroup$
    – Omega Krypton
    May 16 at 4:09






  • 1




    $begingroup$
    Technically, you don't even need trial and error, as rot13(trareny zntavghqr pnyphyngvbaf zrna vg pbhyq bayl or 10. 20 znxrf bar grez 108^5, juvpu vf zber gura 9 qvtvgf).
    $endgroup$
    – Aranlyde
    May 16 at 4:25


















5












$begingroup$

Without computers or calculators (at least until the very end), the answer is




$ X = boxed10 $




The key here is to realize that




the units digit of $ Y $ being 3 limits our possibilities for $ X $ by a lot. Finding the last digit of a positive integer is the same as taking the integer modulo 10, so we will take the given expression for $ Y $ modulo 10, set it equal to 3, and solve for $ X. $




To do this, we




apply Euler's Theorem, which states that for all coprime integers $ a, n $ we have $ a^phi(n) equiv 1 ! pmodn, $ where $ phi(n) $ is Euler's totient function. For this problem, we'll rely on a similar equation $ a^phi(n) + 1 equiv a ! pmodn, $ which works for any integers $ a, n, $ not just coprime.




Applying this theorem:




We have $ n = 10, $ so $$ a^phi(10) + 1 equiv a^5 equiv a ! ! ! pmod10. $$ Thus, $$ begingather* (X - 1)^5 + (X + 7)^5 + (2X + 6)^5 + (4X + 3)^5 + (5X + 8)^5 equiv 3 ! ! ! pmod10 \ (X - 1) + (X + 7) + (2X + 6) + (4X + 3) + (5X + 8) equiv 3 ! ! ! pmod10 \ 13X + 23 equiv 3 ! ! ! pmod10 \ 3X equiv 0 ! ! ! pmod10 \ X equiv 0 ! ! ! pmod10 endgather* $$




Final answer:




We know now that $ X equiv 0 ! pmod10 $ i.e. $ X $ is a multiple of 10 (0, 10, 20, 30, ...). Note that $ Y $ has exactly 9 digits, so $ X = 0$ can be ruled out since the sum will be less than 5 orders of magnitude. $ X = 10, $ however, does have the potential to come close, and by using a calculator we find that indeed it does work. Any higher values of $ X $ would cause it to have more than 9 digits, so this is our final and only answer.




For the record, the final solution for $ Y $ is




$ 816725493 = 9^5 + 17^5 + 26^5 + 43^5 + 58^5 $







share|improve this answer











$endgroup$












  • $begingroup$
    sorry, ninja-ed you, have an upvote!
    $endgroup$
    – Omega Krypton
    May 16 at 4:11










  • $begingroup$
    @OmegaKrypton Well you did get a five minute head start.. I was still typing out my MathJax when your answer popped up! No matters, the OP will decide who gets the check mark :)
    $endgroup$
    – PiIsNot3
    May 16 at 4:16



















5












$begingroup$

So this puzzle hinges on the fact that




$X^5mod10 = X$. ($1^5=1$, $2^5=32$, $3^5=243$, and so on.)




This means that




$(X+9)mod10+(X+7)mod10+(2X+6)mod10+(4X+3)mod10+(5X+8)mod10 = 3mod 10$ (as $(X-1)mod10 = (X+9)mod10$).




This simplifies to




$(13X+33)mod10=3$. Because of how the 3-times table works, this only works if $Xmod10=0$.




Looking only at




number of digits (for order-of-magnitude calculation), $8^5$ (for $X=0$), doesn't work, but $58^5$ (for $X=10$) does, thus making it the only possible solution.




The number is therefore




$816725943$.







share|improve this answer











$endgroup$












  • $begingroup$
    sorry, ninja-ed you, have an upvote!
    $endgroup$
    – Omega Krypton
    May 16 at 4:11











Your Answer








StackExchange.ready(function()
var channelOptions =
tags: "".split(" "),
id: "559"
;
initTagRenderer("".split(" "), "".split(" "), channelOptions);

StackExchange.using("externalEditor", function()
// Have to fire editor after snippets, if snippets enabled
if (StackExchange.settings.snippets.snippetsEnabled)
StackExchange.using("snippets", function()
createEditor();
);

else
createEditor();

);

function createEditor()
StackExchange.prepareEditor(
heartbeatType: 'answer',
autoActivateHeartbeat: false,
convertImagesToLinks: false,
noModals: true,
showLowRepImageUploadWarning: true,
reputationToPostImages: null,
bindNavPrevention: true,
postfix: "",
imageUploader:
brandingHtml: "Powered by u003ca class="icon-imgur-white" href="https://imgur.com/"u003eu003c/au003e",
contentPolicyHtml: "User contributions licensed under u003ca href="https://creativecommons.org/licenses/by-sa/3.0/"u003ecc by-sa 3.0 with attribution requiredu003c/au003e u003ca href="https://stackoverflow.com/legal/content-policy"u003e(content policy)u003c/au003e",
allowUrls: true
,
noCode: true, onDemand: true,
discardSelector: ".discard-answer"
,immediatelyShowMarkdownHelp:true
);



);













draft saved

draft discarded


















StackExchange.ready(
function ()
StackExchange.openid.initPostLogin('.new-post-login', 'https%3a%2f%2fpuzzling.stackexchange.com%2fquestions%2f84005%2ffive-powers-of-fives-produce-unique-pandigital-number-solve-for-x-tell-me-y%23new-answer', 'question_page');

);

Post as a guest















Required, but never shown

























3 Answers
3






active

oldest

votes








3 Answers
3






active

oldest

votes









active

oldest

votes






active

oldest

votes









7












$begingroup$

OK, now i realise its beauty...




for all $i$ from $0$ to $9$, $i^5$ mod $10$ = $i$




so to simply get the ending digit of $X$, the equation can be simplified:




$Y = (X-1) + ( X + 7 ) + ( 2X + 6 ) + ( 4X + 3 ) + ( 5 X + 8) $
$Y=13X +23$




More mod10-ing:




$Y=3X+3$




Sub $Y mod10 = 3$:




$3=3X+3$




Deducing $X mod10$:




$3X=0$
$X=0 (mod 10)$




Then, start from $X = $




$0$




Result (using a calculator in this very last step)




$57593$ Too small...




Next attempt: $X =$




$10$




Result (using a calculator in this very last step)




$Y=816725493$ Nice!




The result above is a pan-digital number as required by OP, so this is done!






share|improve this answer











$endgroup$












  • $begingroup$
    ...yes.........
    $endgroup$
    – Uvc
    May 16 at 4:08










  • $begingroup$
    beautiful question!!! @Uvc +1ed
    $endgroup$
    – Omega Krypton
    May 16 at 4:09






  • 1




    $begingroup$
    Technically, you don't even need trial and error, as rot13(trareny zntavghqr pnyphyngvbaf zrna vg pbhyq bayl or 10. 20 znxrf bar grez 108^5, juvpu vf zber gura 9 qvtvgf).
    $endgroup$
    – Aranlyde
    May 16 at 4:25















7












$begingroup$

OK, now i realise its beauty...




for all $i$ from $0$ to $9$, $i^5$ mod $10$ = $i$




so to simply get the ending digit of $X$, the equation can be simplified:




$Y = (X-1) + ( X + 7 ) + ( 2X + 6 ) + ( 4X + 3 ) + ( 5 X + 8) $
$Y=13X +23$




More mod10-ing:




$Y=3X+3$




Sub $Y mod10 = 3$:




$3=3X+3$




Deducing $X mod10$:




$3X=0$
$X=0 (mod 10)$




Then, start from $X = $




$0$




Result (using a calculator in this very last step)




$57593$ Too small...




Next attempt: $X =$




$10$




Result (using a calculator in this very last step)




$Y=816725493$ Nice!




The result above is a pan-digital number as required by OP, so this is done!






share|improve this answer











$endgroup$












  • $begingroup$
    ...yes.........
    $endgroup$
    – Uvc
    May 16 at 4:08










  • $begingroup$
    beautiful question!!! @Uvc +1ed
    $endgroup$
    – Omega Krypton
    May 16 at 4:09






  • 1




    $begingroup$
    Technically, you don't even need trial and error, as rot13(trareny zntavghqr pnyphyngvbaf zrna vg pbhyq bayl or 10. 20 znxrf bar grez 108^5, juvpu vf zber gura 9 qvtvgf).
    $endgroup$
    – Aranlyde
    May 16 at 4:25













7












7








7





$begingroup$

OK, now i realise its beauty...




for all $i$ from $0$ to $9$, $i^5$ mod $10$ = $i$




so to simply get the ending digit of $X$, the equation can be simplified:




$Y = (X-1) + ( X + 7 ) + ( 2X + 6 ) + ( 4X + 3 ) + ( 5 X + 8) $
$Y=13X +23$




More mod10-ing:




$Y=3X+3$




Sub $Y mod10 = 3$:




$3=3X+3$




Deducing $X mod10$:




$3X=0$
$X=0 (mod 10)$




Then, start from $X = $




$0$




Result (using a calculator in this very last step)




$57593$ Too small...




Next attempt: $X =$




$10$




Result (using a calculator in this very last step)




$Y=816725493$ Nice!




The result above is a pan-digital number as required by OP, so this is done!






share|improve this answer











$endgroup$



OK, now i realise its beauty...




for all $i$ from $0$ to $9$, $i^5$ mod $10$ = $i$




so to simply get the ending digit of $X$, the equation can be simplified:




$Y = (X-1) + ( X + 7 ) + ( 2X + 6 ) + ( 4X + 3 ) + ( 5 X + 8) $
$Y=13X +23$




More mod10-ing:




$Y=3X+3$




Sub $Y mod10 = 3$:




$3=3X+3$




Deducing $X mod10$:




$3X=0$
$X=0 (mod 10)$




Then, start from $X = $




$0$




Result (using a calculator in this very last step)




$57593$ Too small...




Next attempt: $X =$




$10$




Result (using a calculator in this very last step)




$Y=816725493$ Nice!




The result above is a pan-digital number as required by OP, so this is done!







share|improve this answer














share|improve this answer



share|improve this answer








edited May 16 at 10:01

























answered May 16 at 4:01









Omega KryptonOmega Krypton

6,6262953




6,6262953











  • $begingroup$
    ...yes.........
    $endgroup$
    – Uvc
    May 16 at 4:08










  • $begingroup$
    beautiful question!!! @Uvc +1ed
    $endgroup$
    – Omega Krypton
    May 16 at 4:09






  • 1




    $begingroup$
    Technically, you don't even need trial and error, as rot13(trareny zntavghqr pnyphyngvbaf zrna vg pbhyq bayl or 10. 20 znxrf bar grez 108^5, juvpu vf zber gura 9 qvtvgf).
    $endgroup$
    – Aranlyde
    May 16 at 4:25
















  • $begingroup$
    ...yes.........
    $endgroup$
    – Uvc
    May 16 at 4:08










  • $begingroup$
    beautiful question!!! @Uvc +1ed
    $endgroup$
    – Omega Krypton
    May 16 at 4:09






  • 1




    $begingroup$
    Technically, you don't even need trial and error, as rot13(trareny zntavghqr pnyphyngvbaf zrna vg pbhyq bayl or 10. 20 znxrf bar grez 108^5, juvpu vf zber gura 9 qvtvgf).
    $endgroup$
    – Aranlyde
    May 16 at 4:25















$begingroup$
...yes.........
$endgroup$
– Uvc
May 16 at 4:08




$begingroup$
...yes.........
$endgroup$
– Uvc
May 16 at 4:08












$begingroup$
beautiful question!!! @Uvc +1ed
$endgroup$
– Omega Krypton
May 16 at 4:09




$begingroup$
beautiful question!!! @Uvc +1ed
$endgroup$
– Omega Krypton
May 16 at 4:09




1




1




$begingroup$
Technically, you don't even need trial and error, as rot13(trareny zntavghqr pnyphyngvbaf zrna vg pbhyq bayl or 10. 20 znxrf bar grez 108^5, juvpu vf zber gura 9 qvtvgf).
$endgroup$
– Aranlyde
May 16 at 4:25




$begingroup$
Technically, you don't even need trial and error, as rot13(trareny zntavghqr pnyphyngvbaf zrna vg pbhyq bayl or 10. 20 znxrf bar grez 108^5, juvpu vf zber gura 9 qvtvgf).
$endgroup$
– Aranlyde
May 16 at 4:25











5












$begingroup$

Without computers or calculators (at least until the very end), the answer is




$ X = boxed10 $




The key here is to realize that




the units digit of $ Y $ being 3 limits our possibilities for $ X $ by a lot. Finding the last digit of a positive integer is the same as taking the integer modulo 10, so we will take the given expression for $ Y $ modulo 10, set it equal to 3, and solve for $ X. $




To do this, we




apply Euler's Theorem, which states that for all coprime integers $ a, n $ we have $ a^phi(n) equiv 1 ! pmodn, $ where $ phi(n) $ is Euler's totient function. For this problem, we'll rely on a similar equation $ a^phi(n) + 1 equiv a ! pmodn, $ which works for any integers $ a, n, $ not just coprime.




Applying this theorem:




We have $ n = 10, $ so $$ a^phi(10) + 1 equiv a^5 equiv a ! ! ! pmod10. $$ Thus, $$ begingather* (X - 1)^5 + (X + 7)^5 + (2X + 6)^5 + (4X + 3)^5 + (5X + 8)^5 equiv 3 ! ! ! pmod10 \ (X - 1) + (X + 7) + (2X + 6) + (4X + 3) + (5X + 8) equiv 3 ! ! ! pmod10 \ 13X + 23 equiv 3 ! ! ! pmod10 \ 3X equiv 0 ! ! ! pmod10 \ X equiv 0 ! ! ! pmod10 endgather* $$




Final answer:




We know now that $ X equiv 0 ! pmod10 $ i.e. $ X $ is a multiple of 10 (0, 10, 20, 30, ...). Note that $ Y $ has exactly 9 digits, so $ X = 0$ can be ruled out since the sum will be less than 5 orders of magnitude. $ X = 10, $ however, does have the potential to come close, and by using a calculator we find that indeed it does work. Any higher values of $ X $ would cause it to have more than 9 digits, so this is our final and only answer.




For the record, the final solution for $ Y $ is




$ 816725493 = 9^5 + 17^5 + 26^5 + 43^5 + 58^5 $







share|improve this answer











$endgroup$












  • $begingroup$
    sorry, ninja-ed you, have an upvote!
    $endgroup$
    – Omega Krypton
    May 16 at 4:11










  • $begingroup$
    @OmegaKrypton Well you did get a five minute head start.. I was still typing out my MathJax when your answer popped up! No matters, the OP will decide who gets the check mark :)
    $endgroup$
    – PiIsNot3
    May 16 at 4:16
















5












$begingroup$

Without computers or calculators (at least until the very end), the answer is




$ X = boxed10 $




The key here is to realize that




the units digit of $ Y $ being 3 limits our possibilities for $ X $ by a lot. Finding the last digit of a positive integer is the same as taking the integer modulo 10, so we will take the given expression for $ Y $ modulo 10, set it equal to 3, and solve for $ X. $




To do this, we




apply Euler's Theorem, which states that for all coprime integers $ a, n $ we have $ a^phi(n) equiv 1 ! pmodn, $ where $ phi(n) $ is Euler's totient function. For this problem, we'll rely on a similar equation $ a^phi(n) + 1 equiv a ! pmodn, $ which works for any integers $ a, n, $ not just coprime.




Applying this theorem:




We have $ n = 10, $ so $$ a^phi(10) + 1 equiv a^5 equiv a ! ! ! pmod10. $$ Thus, $$ begingather* (X - 1)^5 + (X + 7)^5 + (2X + 6)^5 + (4X + 3)^5 + (5X + 8)^5 equiv 3 ! ! ! pmod10 \ (X - 1) + (X + 7) + (2X + 6) + (4X + 3) + (5X + 8) equiv 3 ! ! ! pmod10 \ 13X + 23 equiv 3 ! ! ! pmod10 \ 3X equiv 0 ! ! ! pmod10 \ X equiv 0 ! ! ! pmod10 endgather* $$




Final answer:




We know now that $ X equiv 0 ! pmod10 $ i.e. $ X $ is a multiple of 10 (0, 10, 20, 30, ...). Note that $ Y $ has exactly 9 digits, so $ X = 0$ can be ruled out since the sum will be less than 5 orders of magnitude. $ X = 10, $ however, does have the potential to come close, and by using a calculator we find that indeed it does work. Any higher values of $ X $ would cause it to have more than 9 digits, so this is our final and only answer.




For the record, the final solution for $ Y $ is




$ 816725493 = 9^5 + 17^5 + 26^5 + 43^5 + 58^5 $







share|improve this answer











$endgroup$












  • $begingroup$
    sorry, ninja-ed you, have an upvote!
    $endgroup$
    – Omega Krypton
    May 16 at 4:11










  • $begingroup$
    @OmegaKrypton Well you did get a five minute head start.. I was still typing out my MathJax when your answer popped up! No matters, the OP will decide who gets the check mark :)
    $endgroup$
    – PiIsNot3
    May 16 at 4:16














5












5








5





$begingroup$

Without computers or calculators (at least until the very end), the answer is




$ X = boxed10 $




The key here is to realize that




the units digit of $ Y $ being 3 limits our possibilities for $ X $ by a lot. Finding the last digit of a positive integer is the same as taking the integer modulo 10, so we will take the given expression for $ Y $ modulo 10, set it equal to 3, and solve for $ X. $




To do this, we




apply Euler's Theorem, which states that for all coprime integers $ a, n $ we have $ a^phi(n) equiv 1 ! pmodn, $ where $ phi(n) $ is Euler's totient function. For this problem, we'll rely on a similar equation $ a^phi(n) + 1 equiv a ! pmodn, $ which works for any integers $ a, n, $ not just coprime.




Applying this theorem:




We have $ n = 10, $ so $$ a^phi(10) + 1 equiv a^5 equiv a ! ! ! pmod10. $$ Thus, $$ begingather* (X - 1)^5 + (X + 7)^5 + (2X + 6)^5 + (4X + 3)^5 + (5X + 8)^5 equiv 3 ! ! ! pmod10 \ (X - 1) + (X + 7) + (2X + 6) + (4X + 3) + (5X + 8) equiv 3 ! ! ! pmod10 \ 13X + 23 equiv 3 ! ! ! pmod10 \ 3X equiv 0 ! ! ! pmod10 \ X equiv 0 ! ! ! pmod10 endgather* $$




Final answer:




We know now that $ X equiv 0 ! pmod10 $ i.e. $ X $ is a multiple of 10 (0, 10, 20, 30, ...). Note that $ Y $ has exactly 9 digits, so $ X = 0$ can be ruled out since the sum will be less than 5 orders of magnitude. $ X = 10, $ however, does have the potential to come close, and by using a calculator we find that indeed it does work. Any higher values of $ X $ would cause it to have more than 9 digits, so this is our final and only answer.




For the record, the final solution for $ Y $ is




$ 816725493 = 9^5 + 17^5 + 26^5 + 43^5 + 58^5 $







share|improve this answer











$endgroup$



Without computers or calculators (at least until the very end), the answer is




$ X = boxed10 $




The key here is to realize that




the units digit of $ Y $ being 3 limits our possibilities for $ X $ by a lot. Finding the last digit of a positive integer is the same as taking the integer modulo 10, so we will take the given expression for $ Y $ modulo 10, set it equal to 3, and solve for $ X. $




To do this, we




apply Euler's Theorem, which states that for all coprime integers $ a, n $ we have $ a^phi(n) equiv 1 ! pmodn, $ where $ phi(n) $ is Euler's totient function. For this problem, we'll rely on a similar equation $ a^phi(n) + 1 equiv a ! pmodn, $ which works for any integers $ a, n, $ not just coprime.




Applying this theorem:




We have $ n = 10, $ so $$ a^phi(10) + 1 equiv a^5 equiv a ! ! ! pmod10. $$ Thus, $$ begingather* (X - 1)^5 + (X + 7)^5 + (2X + 6)^5 + (4X + 3)^5 + (5X + 8)^5 equiv 3 ! ! ! pmod10 \ (X - 1) + (X + 7) + (2X + 6) + (4X + 3) + (5X + 8) equiv 3 ! ! ! pmod10 \ 13X + 23 equiv 3 ! ! ! pmod10 \ 3X equiv 0 ! ! ! pmod10 \ X equiv 0 ! ! ! pmod10 endgather* $$




Final answer:




We know now that $ X equiv 0 ! pmod10 $ i.e. $ X $ is a multiple of 10 (0, 10, 20, 30, ...). Note that $ Y $ has exactly 9 digits, so $ X = 0$ can be ruled out since the sum will be less than 5 orders of magnitude. $ X = 10, $ however, does have the potential to come close, and by using a calculator we find that indeed it does work. Any higher values of $ X $ would cause it to have more than 9 digits, so this is our final and only answer.




For the record, the final solution for $ Y $ is




$ 816725493 = 9^5 + 17^5 + 26^5 + 43^5 + 58^5 $








share|improve this answer














share|improve this answer



share|improve this answer








edited May 16 at 4:19

























answered May 16 at 4:10









PiIsNot3PiIsNot3

4,5181154




4,5181154











  • $begingroup$
    sorry, ninja-ed you, have an upvote!
    $endgroup$
    – Omega Krypton
    May 16 at 4:11










  • $begingroup$
    @OmegaKrypton Well you did get a five minute head start.. I was still typing out my MathJax when your answer popped up! No matters, the OP will decide who gets the check mark :)
    $endgroup$
    – PiIsNot3
    May 16 at 4:16

















  • $begingroup$
    sorry, ninja-ed you, have an upvote!
    $endgroup$
    – Omega Krypton
    May 16 at 4:11










  • $begingroup$
    @OmegaKrypton Well you did get a five minute head start.. I was still typing out my MathJax when your answer popped up! No matters, the OP will decide who gets the check mark :)
    $endgroup$
    – PiIsNot3
    May 16 at 4:16
















$begingroup$
sorry, ninja-ed you, have an upvote!
$endgroup$
– Omega Krypton
May 16 at 4:11




$begingroup$
sorry, ninja-ed you, have an upvote!
$endgroup$
– Omega Krypton
May 16 at 4:11












$begingroup$
@OmegaKrypton Well you did get a five minute head start.. I was still typing out my MathJax when your answer popped up! No matters, the OP will decide who gets the check mark :)
$endgroup$
– PiIsNot3
May 16 at 4:16





$begingroup$
@OmegaKrypton Well you did get a five minute head start.. I was still typing out my MathJax when your answer popped up! No matters, the OP will decide who gets the check mark :)
$endgroup$
– PiIsNot3
May 16 at 4:16












5












$begingroup$

So this puzzle hinges on the fact that




$X^5mod10 = X$. ($1^5=1$, $2^5=32$, $3^5=243$, and so on.)




This means that




$(X+9)mod10+(X+7)mod10+(2X+6)mod10+(4X+3)mod10+(5X+8)mod10 = 3mod 10$ (as $(X-1)mod10 = (X+9)mod10$).




This simplifies to




$(13X+33)mod10=3$. Because of how the 3-times table works, this only works if $Xmod10=0$.




Looking only at




number of digits (for order-of-magnitude calculation), $8^5$ (for $X=0$), doesn't work, but $58^5$ (for $X=10$) does, thus making it the only possible solution.




The number is therefore




$816725943$.







share|improve this answer











$endgroup$












  • $begingroup$
    sorry, ninja-ed you, have an upvote!
    $endgroup$
    – Omega Krypton
    May 16 at 4:11















5












$begingroup$

So this puzzle hinges on the fact that




$X^5mod10 = X$. ($1^5=1$, $2^5=32$, $3^5=243$, and so on.)




This means that




$(X+9)mod10+(X+7)mod10+(2X+6)mod10+(4X+3)mod10+(5X+8)mod10 = 3mod 10$ (as $(X-1)mod10 = (X+9)mod10$).




This simplifies to




$(13X+33)mod10=3$. Because of how the 3-times table works, this only works if $Xmod10=0$.




Looking only at




number of digits (for order-of-magnitude calculation), $8^5$ (for $X=0$), doesn't work, but $58^5$ (for $X=10$) does, thus making it the only possible solution.




The number is therefore




$816725943$.







share|improve this answer











$endgroup$












  • $begingroup$
    sorry, ninja-ed you, have an upvote!
    $endgroup$
    – Omega Krypton
    May 16 at 4:11













5












5








5





$begingroup$

So this puzzle hinges on the fact that




$X^5mod10 = X$. ($1^5=1$, $2^5=32$, $3^5=243$, and so on.)




This means that




$(X+9)mod10+(X+7)mod10+(2X+6)mod10+(4X+3)mod10+(5X+8)mod10 = 3mod 10$ (as $(X-1)mod10 = (X+9)mod10$).




This simplifies to




$(13X+33)mod10=3$. Because of how the 3-times table works, this only works if $Xmod10=0$.




Looking only at




number of digits (for order-of-magnitude calculation), $8^5$ (for $X=0$), doesn't work, but $58^5$ (for $X=10$) does, thus making it the only possible solution.




The number is therefore




$816725943$.







share|improve this answer











$endgroup$



So this puzzle hinges on the fact that




$X^5mod10 = X$. ($1^5=1$, $2^5=32$, $3^5=243$, and so on.)




This means that




$(X+9)mod10+(X+7)mod10+(2X+6)mod10+(4X+3)mod10+(5X+8)mod10 = 3mod 10$ (as $(X-1)mod10 = (X+9)mod10$).




This simplifies to




$(13X+33)mod10=3$. Because of how the 3-times table works, this only works if $Xmod10=0$.




Looking only at




number of digits (for order-of-magnitude calculation), $8^5$ (for $X=0$), doesn't work, but $58^5$ (for $X=10$) does, thus making it the only possible solution.




The number is therefore




$816725943$.








share|improve this answer














share|improve this answer



share|improve this answer








edited May 16 at 4:29

























answered May 16 at 4:10









AranlydeAranlyde

907213




907213











  • $begingroup$
    sorry, ninja-ed you, have an upvote!
    $endgroup$
    – Omega Krypton
    May 16 at 4:11
















  • $begingroup$
    sorry, ninja-ed you, have an upvote!
    $endgroup$
    – Omega Krypton
    May 16 at 4:11















$begingroup$
sorry, ninja-ed you, have an upvote!
$endgroup$
– Omega Krypton
May 16 at 4:11




$begingroup$
sorry, ninja-ed you, have an upvote!
$endgroup$
– Omega Krypton
May 16 at 4:11

















draft saved

draft discarded
















































Thanks for contributing an answer to Puzzling Stack Exchange!


  • Please be sure to answer the question. Provide details and share your research!

But avoid


  • Asking for help, clarification, or responding to other answers.

  • Making statements based on opinion; back them up with references or personal experience.

Use MathJax to format equations. MathJax reference.


To learn more, see our tips on writing great answers.




draft saved


draft discarded














StackExchange.ready(
function ()
StackExchange.openid.initPostLogin('.new-post-login', 'https%3a%2f%2fpuzzling.stackexchange.com%2fquestions%2f84005%2ffive-powers-of-fives-produce-unique-pandigital-number-solve-for-x-tell-me-y%23new-answer', 'question_page');

);

Post as a guest















Required, but never shown





















































Required, but never shown














Required, but never shown












Required, but never shown







Required, but never shown

































Required, but never shown














Required, but never shown












Required, but never shown







Required, but never shown







Popular posts from this blog

Get product attribute by attribute group code in magento 2get product attribute by product attribute group in magento 2Magento 2 Log Bundle Product Data in List Page?How to get all product attribute of a attribute group of Default attribute set?Magento 2.1 Create a filter in the product grid by new attributeMagento 2 : Get Product Attribute values By GroupMagento 2 How to get all existing values for one attributeMagento 2 get custom attribute of a single product inside a pluginMagento 2.3 How to get all the Multi Source Inventory (MSI) locations collection in custom module?Magento2: how to develop rest API to get new productsGet product attribute by attribute group code ( [attribute_group_code] ) in magento 2

Category:9 (number) SubcategoriesMedia in category "9 (number)"Navigation menuUpload mediaGND ID: 4485639-8Library of Congress authority ID: sh85091979ReasonatorScholiaStatistics

Magento 2.3: How do i solve this, Not registered handle, on custom form?How can i rewrite TierPrice Block in Magento2magento 2 captcha not rendering if I override layout xmlmain.CRITICAL: Plugin class doesn't existMagento 2 : Problem while adding custom button order view page?Magento 2.2.5: Overriding Admin Controller sales/orderMagento 2.2.5: Add, Update and Delete existing products Custom OptionsMagento 2.3 : File Upload issue in UI Component FormMagento2 Not registered handleHow to configured Form Builder Js in my custom magento 2.3.0 module?Magento 2.3. How to create image upload field in an admin form